Preview (15 of 111 pages)

NCLEX UWorld 2023 Real Exam Questions (1,819 Terms) All Correctly
Answered, A+ Score Solution
Part 1
When do advanced directives go into effect?
Answer: when person is unable to speak for him/herself due to either:
1. Mental Incapacity - coma (GCS score ≤ 7)
2. Aphasia
(≠as soon as signed; directives can always be changed later by person)
SBAR Communication Framework Components
Answer: 1. S = Situation - what prompted the communication (eg what changes occurred)
2. B = Background - pertinent information, relevant history, vital signs
3. A = Assessment - nurse's assessment of the situation (when & what changes occurred)
4. R = Recommendation - request for prescription or action from HCP
Appropriate order of actions when client found on floor
Answer: 1. Assessment of physiological stability (ABCs)
2. Assessment of injuries
3. Moving client
4. Notifications
5. Documentation
Conditions of being ineligible to leave AMA
Answer: 1. danger to self or others
2. lack of consciousness
3. Altered consciousness
4. Mental illness
5. Being under chemical influence
6. Court decision
Effective handoff communication components

Answer: Nurse should:
1. Provide identifying information (eg client's name and room number)
2. Note care priorities and upcoming or outstanding tasks (eg time to replace medication
infusion bag, need to perform delayed wound care and cause of delay)
3. Provide exact, pertinent information (eg medication dose, time, measurable outcomes)
4. Include multidisciplinary plans (eg radiology examinations, family meetings, physical
therapy)
5. Relay significant client changes in a clear manner
Risk factors for cervical cancer
Answer: 1. Infection with high-risk HPV strains
2. History of sexually transmitted diseases
3. Early onset of sexual activity
4. Multiple or high-risk sexual partners
5. Immunosuppression
6. Oral contraceptive use
7. Low SES
8. Tobacco use
What medications interact with grapefruit?
Answer: 1. calcium channel blockers (diltiazem, nifedipine, verapamil, etc)
2. statins
3. SSRIs
Risk associated with stent placement using the femoral approach
Answer: retroperitoneal haemorrhage
What are early signs of bleeding into the retroperitoneal space?
Answer: hypotension, back pain, flank ecchymosis (grey turner sign), hematoma formation,
diminished distal pulses
What is the grey-turner sign and what is it a sign of?
Answer: bruising of the flanks and retroperitoneal haemorrhage and is a bluish color

What are some physical signs of peripheral arterial disease?
Answer: intermittent calf muscle pain?, rest pain, hair loss, decreased peripheral pulses, cool,
dry, shiny skin, thick brittle nails, gangrene, ulcers (all of these are in the extremities)
Transplanted hearts are expected to be
Answer: tachycardic like 90-110
What is the priority intervention for pain with sickle cell crisis and why?
Answer: administer IV fluids to reduce blood viscosity and restore perfusion to areas
affected by Vaso occlusion
What is the purpose of continuous bladder irrigation?
Answer: it is prescribed after TURP to prevent obstruction of urine outflow by removing
clotted blood from the bladder
What is the nurses care of monitoring CBI?
Answer: monitor quality of drainage, titrate the inflow rate, and manually irrigating as
needed
Characteristics of a basilar skull fracture
Answer: Periorbital hematomas (raccoon eyes), CSF fluid rhinorrhoea, and Battle sign
(bruising behind the ear).
Immediate client care for basilar skull fracture
Answer: cervical spine immobilization, close neurologic monitoring, and support of ABCs
Vomiting with intake may mean
Answer: viral or bacterial infection
Tympanostomy tubes are placed for
Answer: Recurrent otitis media.
Nurse actions during a seizure

Answer: Assist them to lie down if standing or sitting, place them on their side for a patent
airway, loosen tight clothing, give oxygen as needed, remove objects from the immediate
area, document the time and duration of the seizure (for tests are done later to see which type
of seizure and maybe what exacerbates it); never put anything in their mouth or restrain them,
as muscle contractions can occur during a seizure.
What are some early symptoms of ICP?
Answer: Altered LOC, headache, abnormal breathing, rise in BP, slow pulse, vomiting.
Client who has a TIA is often placed on
Answer: prophylactic antithrombotic treatment like aspirin or clopidogrel
Glasgow Coma Scale ranges from
Answer: 3-15; 3 being worst 15 being best condition (8 or below in a coma)
What are the 3 components?
Answer: • eye opening
• motor response
• verbal response
What is a primary component in TPN?
Answer: glucose, so the nurse should be monitoring blood glucose and be assessing for signs
of hyperglycemia
When a client is on TPN, the nurse must assess for hyperglycemia why?
Answer: Because a primary component is glucose; therefore, the nurse must evaluate if the
client is receiving too much glucose (hyperglycemia). Additionally, a large urinary output,
such as 4800 mL, could indicate symptoms of hyperglycemia.
Signs of hyperglycemia
Answer: • polydipsia
• Polyuria
• Restlessness
• Confusion

• Blood glucose over 200
• Fatigue
• Headache
• Blurred vision
• Kussmaul respirations.
Interventions to resolve TPN-associated hyperglycemia
Answer: • reduce amount of carbohydrate in TPN solution
• slow down infusion rate
• administer subcutaneous insulin
What is the goal for mass casualty events?
Answer: do the greatest good for the greatest number of people
Keep in mind that disaster triage ranks the likelihood of survival with treatment, not
necessarily the severity of the injury
Answer: Disaster triage prioritizes patients based on their survival potential with treatment,
rather than the severity of their injuries. This approach ensures that resources are allocated to
those most likely to benefit from them.
What are the 4 categories for triaging?
Answer: immediate (red tag)- life threatening injuries with good prognosis once treated
delayed (yellow)- injuries requiring treatment within hours minimal (green tag)- injuries
requiring treatment within a few days expectant (black tag)- extensive injuries, poor
prognosis regardless of treatment
Rule of nines
Answer: head: 4.5 front 4.5 back torso: 18 front 18 back each arm: 4.5 front 4.5 back (each
arm is 9 total) each leg: 9 front 8 back (each is 18 total) genitals: 1
Extrapyramidal side effects
Answer: • Acute Dystonic
• Reaction: sudden onset sustained muscle contractions
• Akathisia: restlessness with inability to sit still drug induce

• Parkinsonism: tremor, rigidity, bradykinesia, masked like faces
• Akinisia: loss of involuntary movement
• Tardive Dyskinesia
• Neuroleptic Malignant Syndrome
Don’t give morphine if RR under 12 because it can cause
Answer: respiratory depression
Airborne precautions
Answer: • tuberculosis, varicella, and rubeola (measles)
• wear N95 respirator (and other as needed like for splashes)
• Also (neg pressure room and HEPA)
• clients suspected are to wear a surgical mask after triage
UAP soft wrist restraints can:
Answer: • do ROM exercises
• reapply wrist restraints
• report changes in skin to nurse
• turn/reposition client in bed
A client with major depression and severe weight loss needs what type of diet?
Answer: High in calories and protein, also foods that are easy to chew and don’t require a lot
of energy because they may have a low energy level. Examples: while milk/dairy, granola
muffins, potatoes, meat fish eggs, pasta also small frequent meals
Memorize MAOIs and remember
Answer: they can’t eat foods high in tyramine like aged cheese, yogurt, fermented foods,
beer, red wine, chocolate, avocados
Do not give Lasix to a pneumonia patient with fine crackles because they do not result from
heart failure or edema.
Answer: Do not administer Lasix to a pneumonia patient with fine crackles, as they are not
indicative of heart failure or edema.

You can give pneumonia patients expectorants, antibiotics, mucolytics, antipyretics,
analgesics, and anti-inflammatories; examples include
Answer: Mucinex, Ibuprofen
Decerebrate posturing (toes point down and arm’s/legs straight out) is a sign of
Answer: severe brain damage
Near drowning hypothermia
Answer: warm iv fluids, blankets, and air also will find weak and thready pulse
A client is not dead until warm and dead!!!
Answer: sometimes a patient is so cold that a pulse cannot be felt intussusception
Causes intestinal obstruction
Answer: ileum telescopes into cecum, pain obstruction, edema, compression of BVs, bowel
ischemia, rectal bleeding (currant jelly stools)
Signs of intussusception
Answer: initial periodic pain with legs drawn up to abdomen, pain is severe and progressive
though, inconsolable crying, blood/mucous stools "current jelly"
How is it treated?
Answer: an air enema
Guaifenesin (Mucinex)
Answer: Expectorant medication that increases respiratory fluids and thins secretions to
facilitate expectoration.
Erythropoietin (Epogen)
Answer: stimulates bone marrow to make RBCs and combats the effects of chemo and used
for kidney disease
When is hemoccult done?
Answer: when hidden (occult) blood is suspected due to dark and tarry stool

Treat intussusception with
Answer: Air Enema
Tetralogy of Fallot
Answer: • exacerbation can happen when infant or child cries, becomes upset, or is feeding
• you immediately place them in knee chest position
Haemolytic uremic syndrome
Answer: life threatening complication of e. coli diarrhoea
What are the signs?
Answer: anaemia (pallor), low platelets (petechiae and purpura, and acute kidney injury (low
UO)
Cognitive behavioural therapy (CBT) 5 basic components
Answer: • education about pts disorder
• self observing and monitoring
• relaxation techniques
• cognitive restructuring
• behavioural strategies
Specimen collection from a Foley catheter is considered sterile and should not be delegated to
UAP.
Answer: Specimen collection from a Foley catheter is sterile and must not be delegated to
UAP.
A clients first and last name is not PHI privileged health info
Answer: it is not a violation to call them by their first and last names or have info heard
inadvertently
Written consents - Nurse's Role
Answer: 1. Witness that consent was signed voluntarily
2. That patient was competent at time of signing

3. Documenting in medical records after signature obtained with date/time of signature
What happens if a client does not fully understand informed consent?
Answer: the nurse must contact the doctor. the nurse is not responsible for verifying that the
client understands
How to stop epistaxis
Answer: tilt the head forward and apply direct continuous pressure on the aloes (sides) for
about 5-20 mins can also hold a cold washcloth to the bridge of nose for vasoconstriction
keep child calm and quiet
IV iodinated contrast used for client scan can cause
Answer: kidney injury
Metformin is discontinued on the day of IV iodine contrast exposure
Answer: Metformin is discontinued on the day of IV iodine contrast exposure to prevent
potential kidney damage.
Amlodipine
Answer: calcium channel blocker used to treat hypertension
Gabapentin (Neurontin)
Answer: used for neuropathic pain
Phenytoin (Dilantin)
Answer: antiseizure
Glipizide
Answer: oral sulfonylurea controls blood sugar
Levofloxacin (Levaquin)
Answer: antibiotic
Testicular self examination

Answer: • perform monthly and on same day
• perform while taking hot shower
• because temps will relax scrotal tissue and
• make testis hang lower in scrotum
• use both hands to feel each testis separately
• palpate gently using thumb and first 2 fingers
Normal calcium level
Answer: 8.6-10.2
Review heart rhythms
Answer: Reviewing heart rhythms involves analyzing the electrocardiogram (ECG) for
abnormalities in rate, rhythm, and morphology to identify conditions like arrhythmias or
ischemia.
Lactated ringers is often used for burns
Answer: Lactated Ringer's solution is often used for fluid resuscitation in burn patients.
Addisons disease = hyperpigmentation of the skin
Answer: Addison's disease is characterized by hyperpigmentation of the skin due to
increased melanin production.
What is one of the earliest signs of ICP?
Answer: change in responsiveness
What are some expected symptoms in a TBI?
Answer: Low-grade constant headache, slowness in thinking, memory problems, loss of
balance, poor coordination, constant exhaustion, increased sensitivity to light, and heightened
irritability— all symptoms that can last up to 6 weeks.
Bacterial meningitis
Answer: • high fever
• change in loc
• nuchal rigidity

• meningeal signs (positive kernig and brudzinski signs)-- treat with antibiotics
What is the kernig sign?
Answer: patient lies supine, thigh is flexed at right angle, and it hurts to extend leg
What is the brudzinski sign?
Answer: bending of neck causes flexion of knee and hip
What are these 3 signs indicative of?
Answer: meningitis
Client identifiers
Answer: • first and last name
• medical record number
• DOB
Expected term newborn findings
Answer: • plantar creases up
• entire sole
• presence of babinski
• Epstein's pearls
Is the babinski sign present at birth?
Answer: yes but it disappears at 1 year
What does an absent babinski or weak reflex indicate?
Answer: a neurological deficit
Babinski sign for a child less than 1 year
Answer: great toe bends upward and smaller toes fan out. this is normal
Babinski sign for child more than 1 year and an adult
Answer: • plantar flexion
• normal toe flexion (no babinski)

What are epstein’s pearls?
Answer: white pearl like cysts on gum an palate that are benign and usually go away within a
few weeks
When does the umbilical cord detach from body?
Answer: within 2 weeks
Jaundice in a newborn
Answer: • during first 24 hours: jaundice is pathological (related to liver problems)
• after 24 hours: physiological jaundice amount of unconjugated
• bilirubin in system
Infant formula key points
Answer: • never dilute or concentrate formula
• wash tops of formula cans before opening
• unused prepare formula can be used for up to 48 hours and then discarded after
• to warm prepared bottle, place in pan of hot water
• never microwave formula
• any formula left over should be thrown out immediately when done
Long term management of hyper tension
Answer: important to take bp meds as prescribed
What do statin drugs like atorvastatin, rosuvastatin do?
Answer: cut LDL drastically and reduce total cholesterol and triglycerides, and INC HDL
Good cholesterol
Answer: HDL
Bad cholesterol
Answer: LDL
What is a serious complication of statin meds?

Answer: rhabdomyolysis
What is rhabdomyolysis?
Answer: the breakdown of muscle tissue releases muscle fiber contents into the blood. these
substances can cause kidney damage
What are some early signs of rhabdomyolysis?
Answer: muscle aches or weakness- immediately report to HCP!
Behaviors of bulimic person
Answer: • episodes of binge eating
• followed by self-induced vomiting
• using enemas/laxatives
• intense frequent exercise
Signs of bulimia someone may notice
Answer: • going to bathroom after meals
• large amounts of food disappearing
• hidden wrappers/empty food containers like sweets
• intense physical exercise
• parotid gland enlargement
• calluses on hands
• preoccupation with weight food and dieting
Weight loss/gain in anorexic and bulimic patients.
Answer: • anorexic: lose
• weight
• bulemic: maintain weight
Children born before 38 weeks are
Answer: • preterm and must
• "catch up" their developmental milestones
Onset of type 1 diabetes

Answer: childhood
Onset of type 2 diabetes
Answer: after age 40
Medicine healers are important part of native American culture
Answer: be mindful of a patients spiritual needs. allow them to do rituals
First thing to do if a patient is in ventricular tachycardia?
Answer: assess them for a pulse (because they can either have a pulse or not)
How to treat unstable client in V Tach with a pulse?
Answer: synchronized cardioversion
How to treat stable client in V Tach with a pulse?
Answer: Antiarrhythmic medications like amiodarone.
When to initiate CPR and defibrillation with a client in V Tach?
Answer: only if there is no pulse
What is in the LPN scope of practice?
Answer: • monitor RN findings
• reinforce education
• routine procedures like catheterization
• most medication administrations
• ostomy care
• tube patency and enteral feeding
• specific assessments
Notify hcp if child temp is over 100.4°F after immunizations
Answer: Notify the healthcare provider if the child's temperature exceeds 100.4°F after
immunizations.
What are common SE of immunizations?

Answer: • mild fever and soreness and redness at injection site
• anorexia/fussiness in the first 24 hours
GP LLB/LLLA receptor inhibitors (abciximab, etc) can cause serious bleeding
Answer: careful monitoring of puncture site after percutaneous coronary intervention
Signs of internal bleeding and to watch carefully when GP LLB/LLLA receptor inhibitors are
given
Answer: hypotension, tachycardia, changes in heart rhythm, blood in urine, abdomen/back
pain, mental status changes, black tarry stools
Thrombocytopenia
Answer: low platelet count; so this increases risk for bleeding
What is SIADH?
Answer: • high antidiuretic hormone production leads to water retention
• increased total body water
• dilutional hyponatremia
• will see signs of fluid volume overload, changes in loc, weight gain without edema,
hypertension, tachycardia
• seizure precautions
What can hyponatremia cause?
Answer: • Seizures, confusion, neurologic complication
• institute seizure precautions
Treatment of SIADH
Answer: • Fluid restriction to less than 1000 mL per day
• Oral salt tablets to increase serum sodium
• Hypertonic saline (3%) or normal saline (NS) IV and/or vasopressin receptor
• antagonists to decrease renal response to ADH.

So a patient with SIADH

Answer: • does not need extra fluid
• needs salt
• seizure precautions
• strict I/OS
RACE for FIRES in a hospital
Answer: • R: rescue any pts in immediate danger and move them to safety
• A: alarm- sound alarm
• C: confine fire by closing all doors to all rooms
• E: extinguish fire with extinguisher
What is asystole?
Answer: complete absence of ventricular electrical activity in the heart. no ventricular
contraction occurs client is pulseless, apnoeic, and unresponsive
How to treat asystole?
Answer: • CPR
• initiate advanced cardiac life support
• give epinephrine and/or vasopressin
• place an airway
Hepatic encephalopathy
Answer: sleep disturbances (early), lethargy, coma, mental status altered, not oriented to
person place or time, asterixis- flapping tremor of hands, fetor hepaticus, elevated ammonia
Asterixis
Answer: flapping tremors of the hands
How is asterixis assessed?
Answer: have patient extend the arms and dorsiflex the wrist
What is the drug of choice for SVT?
Answer: • adenosine - administer it rapidly over 1-2 secs and flush after with NS
• administer as close to heart as possible (NOT distally)

Before the med, how is SVT usually treated?
Answer: vagal maneuvers (eg bearing down)
What is adenosine?
Answer: • an arrhythmic
• it treats irregular heartbeats
What is the purpose of having a patient do a vagal maneuver?
Answer: The purpose of having a patient perform a vagal maneuver is to slow down a rapid
heart rate by stimulating the vagus nerve.
Full weight bearing
Answer: independent; no assistance needed unless they are uncooperative or they are a fall
risk, which is 1 person standby
Partial weight bearing
Answer: 1. person assist stand & pivot transfer with gait belt or motorized assist device if
cooperative
2. person assist with full body sling if uncooperative
No weight bearing
Answer: 1. motorized assist device if cooperative and they have upper body strength
2. person assist with full body sling if uncooperative and/or has no upper body strength
Clients should use as much of their own weight as possible
Answer: Clients should use as much of their own weight as possible to enhance mobility and
strength during rehabilitation.
How to measure how to safely transfer a patient the first time
Answer: • assess if they can bear weight
• assess whether they are cooperative
1 gram of weight is equal to how many ml of fluid?

Answer: 1
What is adequate UO for an infant?
Answer: 2 ml/kg/hr
How to measure UO in diapers?
Answer: subtract the weight of the diaper when dry from its weight when wet
How often and where is growth hormone replacement given to a child?
Answer: daily sub q injections treatment is most successful when it begins early in a child's
life, as soon as growth delays are noted
When is replacement therapy usually stopped?
Answer: when bone growth stops or when parents determine
What is the classic sign of a tenson pneumothorax?
Answer: mediastinal shift and tracheal deviation
Why does tension pneumothorax cause hypotension?
Answer: Because the heart and great vessels are compressed/shifted and cardiac output is
reduced it is life threatening
What is the treatment for tension pneumothorax?
Answer: • emergency large bore needle decompression
• chest tube
If injury to the spinal cord, what important precaution to take?
Answer: hard cervical collar and backboard
Major depressive disorder demonstrates
Answer: • loss of appetite, weight loss, and insomnia or hypersomnia
• sleep disturbances
How to help improve sleep

Answer: • don’t nap during day
• physical activity at least 5 hours before bed 20 mins of natural sunlight
• avoid caffeine after noon
• avoid alcohol or smoking at bedtime
• relaxing activity before bed
• decrease environmental stimuli
• avoid heavy meals or large amounts of fluid at bedtime
• warm milk or small carbs before bed
What can a baby born to an opioid dependent mom have?
Answer: neonatal abstinence syndrome. the baby experiences opioid withdrawal 24-48 hrs
after birth
What are symptoms of withdrawal from opioid in infants?
Answer: • the baby is hypersensitive
• irritability
• high pitched cry
• jitteriness
• sneezing
• diarrhoea
• vomiting
• poor feeding
• treatment: opioid therapy like methadone or morphine
What is the nursing care focus opioid dependent newborns?
Answer: • reduce stimulation
• promote nutrition and comfort
After feeding an infant, what does placing it in the side lying position do
Answer: promotes gastric emptying and reduces the risk of vomiting
Acceptable methods of blood collection in neonate
Answer: • heel stick

• venipuncture (drawing blood from vein) - considered less painful and often requires fewer
punctures to obtain sample, especially if larger volume is needed
Neonatal heel stick
Answer: - Used to collect a blood sample to assess capillary glucose and perform newborn
screening for inherited disorders (e.g., congenital hypothyroidism, phenylketonuria).
Neonatal heel stick proper technique
Answer: 1. Select location on the medial or lateral side of the outer aspect of the heel
2. Warm the heel for several minutes with warm towel compress or approved single-use
instant heat pack
3. cleanse intended puncture site with alcohol
4. Sucrose and nonnutritive sucking on a pacifier may reduce pain
5. Use an automatic lancet
What is the blood loss amount to be considered postpartum haemorrhage (PPH)?
Answer: • >500 ml after vaginal birth
• >1000 ml after c-section
What is uterine atony?
Answer: a soft boggy poorly contracted uterus and most commonly causes early PPH
What usually causes delayed postpartum haemorrhage?
Answer: retained placental fragments
What are some risk factors for postpartum haemorrhage?
Answer: • A history of PPH
• Uterine distension
• Uterine fatigue
• High parity
• Certain medications.
What do Chvostek’s and Trousseau’s signs indicate?
Answer: Hypocalcaemia.

What to do first with an unrestrained pt involved in a sudden deceleration accident during
emergency trauma care?
Answer: • spinal precautions are first!! before checking pulse and
• opening their airway
• place cervical collar
Dementia characteristic
Answer: • slow onset
• family members usually notice first
• gradual progressive inability to remember recent events
Delerium signs
Answer: • fluctuating acute mental status changes
• inattention with disorganized thinking like hallucinations
• altered level of consciousness and
• the presence of infection can cause delirium
What is a barium enema aka lower GI series?
Answer: • uses fluoroscopy to visualize the colon outlined by contrast to detect
• abnormalities like polyps, ulcers, tumors, and diverticula
Pre-procedure instructions for barium enema
Answer: • take a cathartic (magnesium citrate, polyethylene glycol) to empty stool from the
colon
• CLD day before procedure for bowel prep and to avoid dehydration.
• avoid red/purple liquids
• NPO 8 hours before test
• abdominal cramping may be felt during procedure
Post procedure instructions for barium enema
Answer: • chalky white stool until barium passes
• take a laxative like milk of magnesia to help expel barium
• drink plenty of fluids to promote hydration and eat a high fiber diet.

Make sure directions given from nurse to up are specific and explicit
Answer: Drink 8-10 glasses of water today to help flush out the barium and expect lightcolored stools for a few days. Contact your healthcare provider if you experience severe
abdominal pain, persistent constipation, or signs of infection.
GCS score of 8 or below is classified as what?
Answer: a coma
When you are 8, intubate
Answer: When a patient's age is 8 years or older, intubation may be indicated if they show
signs of respiratory distress or failure.
Amyotrophic lateral sclerosis (ALS) typical symptoms
Answer: limb weakness, dysarthria (difficulty speaking), and dysphagia
What is trismus?
Answer: • Lockjaw
• In ability to open the mouth due to a tonic contraction of the chewing muscles
Scarlett fever
Answer: sandpaper skin rash and fever
Peritonsillar access? a collection of pus
Answer: •a serious complication that can result from tonsilitis
• symptoms include fever, sore throat, dysphagia, trismus, muffled voice, uvula deviation,
pooling of saliva
Normal pH
Answer: 7.35-7.45
PaÓ
Answer: 80-100

PaCÓ
Answer: 35-45
HCǑ
Answer: 22-26
Ó sat
Answer: 95-99
ABG values that mean ARF are what?
Answer: • PaÓ less than or equal to 60 and
• PaCÓ greater than or equal to 50
What type of med is clopidogrel?
Answer: antiplatelet and should be stopped 5-7 days before surgery
NSAIDS should also be stopped 7 days before surgery to prevent excess bleeding
Answer: NSAIDs should be discontinued 7 days before surgery to prevent excess bleeding.
Ginko = Bleeding
Answer: Ginkgo can increase the risk of bleeding due to its anticoagulant properties.
What does foul smelling lochia suggest?
Answer: endometrial infection
What are other signs of endometrial infection?
Answer: maternal fever, tachycardia, and uterine pain/tenderness
WBC is usually elevated up to what during the first 24 hours postpartum?
Answer: up to 30,000. temperature is also elevated
Pinworm infection
Answer: anal itching that’s worse at night

How much folic acid do pregnant women need daily?
Answer: MIN of 400 MCG
What foods have folic acid?
Answer: • green leafy vegetables are an excellent source
• others are beans, rice, peanut butter, and fortified cereals
What is topical capsaicin cream (Zostrix)?
Answer: • OTC analgesic that relived minor pain like osteoarthritis
•special instruction: wait 30 mins after massaging cream into hands before washing
Orthodox jews can’t take capsules why?
Answer: the gelatine is not kosher (fit to be consumed)
What is the most accurate indicator of fluid loss or gain?
Answer: daily weight
What are some risk factors for respiratory depression?
Answer: • elderly
• underlying pulmonary disease
• post-surgery
• obesity
• smoking
• opiate naive
Nursing care for pts in acute mania
Answer: • quiet structured
• non stimulating environment
• one on one activities instead of group
• limiting contact with other people
• high protein high calorie finger foods easy to eat
What type of drug is theophylline
Answer: bronchodilator

Symptoms of theophylline toxicity
Answer: CNS stimulation, such as headache, insomnia, and seizures; nausea/vomiting;
cardiac toxicity.
Digoxin toxicity
Answer: 1st GI disturbances and visual changes
What type of drainage is expected after a surgical procedure?
Answer: serosanguinous (pink)
Normal amount of drainage in first 24 hours following surgery
Answer: depends on type of procedure performed, but about 80-120 mL/hour of
serosanguineous or sanguineous drainage can be expected
What does a CPAP machine do?
Answer: It provides positive pressure to open the airway that was previously cut off
narcotics like morphine can do what with obstructive sleep apnea symptoms
Answer: exacerbate them
Opioid agonist-antagonist meds can be used when during labour?
Answer: in the active phase of stage 1 labour when labour contractions are well established
and the cervix is dilated to at least 4 cm
If pt is in respiratory distress what do you assess first
Answer: Airway
What are the major side effects of opioid analgesics
Answer: sedation, respiratory depression, hypotension, constipation
How should hydromorphone be administered?
Answer: slowly over 2-3 minutes

Intramuscular injections can cause bleeding
Answer: Intramuscular injections can cause bleeding at the injection site.
How should IV potassium be administered?
Answer: must be given in an infusion pump so the rate can be regulated. not by gravity
infusion. it should also always be diluted
Symptoms of cold stress in a newborn
Answer: irritability, lethargy, decreased temp, hypoxia, bradycardia, weak suck and cry,
emesis, hypoglycemia
Whose at a high risk for cold stress and why?
Answer: premature babies due to decreased brown adipose tissue
Do babies shiver to produce heat?
Answer: No
What is Red Man Syndrome and what does it happen from?
Answer: Rapid infusion of vancomycin—flushing, erythema, and pruritus on the face, neck,
and chest.
What is wound evisceration?
Answer: protrusion of internal organs thru the wall of an incision. placing them in low
fowlers with the knees bent helps reduce the tension on the wound
Don’t forget to always assess first
Answer: Always remember to assess first before taking any further action.
What should always be at beside for a gastric lavage?
Answer: intubation and suction supplies
Why is it done and how soon after overdose?
Answer: remove toxins and irrigate stomach and within one hour after overdose

Manifestations of OA
Answer: • pain exacerbated by weight
• bearing
• crepitus
• morning stiffness
• dec joint mobility and rom
• atrophy of the muscles
During pregnancy, why are women in a hypercoagulable state?
Answer: to protect them from haemorrhage but there is a risk for DVT ambulate the pt soon
after childbirth
Who should get the flu vaccine?
Answer: all pts 6 months old or greater unless pt has life threatening allergy to the vaccine or
one of its ingredients
Symptoms of Addisonian crisis
Answer: sudden drop in bp, tachycardia, hypoglycemia, abdomen pain, and weakness
How to treat
Answer: give IV hydrocortisone
Warfarin
Answer: blocks the availability of vitamin k and reduces risk of stroke venous thrombosis or
PE
What is the fluid of choice for the rapid correction of hypotension?
Answer: 0.9 sodium chloride
What is nitro-glycerine used for?
Answer: it’s a vasodilator that restores myocardial perfusion
Signs and symptoms of digoxin toxicity
Answer: • Visual changes

• Nausea/vomiting
• Confusion
• Lethargy
• Weakness
• Cardiac arrhythmias.
Take pulse and with old which med if less than 60?
Answer: digoxin
Antihypertensives are held before dialysis
Answer: Antihypertensives are often held before dialysis to prevent excessive drops in blood
pressure during the procedure.
Never clamp a chest tube during transport of a pt. what may it cause?
Answer: Tension pneumothorax cut the air has no way to escape
What is parallel play that toddlers do?
Answer: play alongside rather than with other children
How to assist a blind pt with walking?
Answer: walk slightly ahead of them and to the side, and let them grab onto your elbow
Post op instructions for total hip replacement
Answer: • avoid adduction and hip flexion
• client supine with hob elevated less than or equal to 45 degree.
What are the steps to deliver a dose via a metered dose inhaler MDI?
Answer: • shake MDI and attach to spacer
• exhale completely
• place lips tightly around mouthpiece
• deliver one puff of med into spacer
• take a slow deep breath and hold it for 10 seconds
• rinse mouth with water

Why is putting an IV in the arm of a mastectomy contraindicated?
Answer: any trauma to the arm on the operative side can cause lymphedema with painful and
lengthy swelling
What are some reasons for venipuncture to be contraindicated un upper extremities?
Answer: • weakness
• paralysis
• infection
• AV fistula or graft
• mastectomy
What are the 2 clinical features of MDD
Answer: depressed mood and loss of interest or pleasure
Why do opioids cause hypotension?
Answer: they dilate peripheral BVs. if the pt stands it can cause orthostatic hypotension
Classic heart attack symptoms
Answer: dizziness, sweating cold clammy skin, sob, nausea/vomiting, ischemic chest pain,
pain radiating to neck, jaw, left shoulder, arms, epigastrium
Symptoms of a pleural effusion
Answer: dyspnea on exertion, non-productive cough, diminished breath sounds, dullness to
percussion, dec tactile fremitus
How soon after a blood transfusion does an acute haemolytic reaction occur?
Answer: 15 mins
What are normal fetal heart tones?
Answer: 110-160
What is nifedipine?
Answer: calcium channel blocker antihypertensive. do not give if bp on the lower end range

What side effects do beta adrenergic have?
Answer: tremor or palpitations
Normal sodium value
Answer: 135-145
If sodium is too low what could this lead to?
Answer: altered mental status and seizures
Cystic fibrosis
Answer: • bronchodilators before chest
• physiotherapy
• diet high in fat calories and proteins
• pancreatic enzyme supplements with all meals and snacks
• recurrent respiratory infections
What can determine if a person with heart problems can resume sexual activity?
Answer: walking 1 block or climbing 2 flights of stairs without symptoms
What is a major complication of RA? morning stiffness. how to fix?
Answer: take a warm shower or bath
Characteristics of neuroleptic malignant syndrome
Answer: fever, muscle rigidity, altered mental status, sweating, hypertension, and tachycardia
What does torsade’s de pointes look like
Answer: a strand of DNA
What is the first line of treatment for torsade’s de pointes?
Answer: IV magnesium
A prolonged QT interval is usually the result of electrolyte imbalances so what is usually the
treatment?
Answer: look for electrolyte in the answer

Drug of choice for symptomatic hypotension?
Answer: dopamine
Tachyarrhythmias?
Answer: metoprolol can be used
What does DOPAMINE do?
Answer: speeds up the heart rate increases cardiac contractility
What type of fluids do pts in hypovolemic shock need?
Answer: isotonic solutions like NS, LR
What is the purpose of norepinephrine?
Answer: vasoconstriction and improved contractility/output
What are some non-pharmacologic interventions that help with phantom limb pain?
Answer: wrapping the extremity or applying ice
What are some signs of a bowel obstruction?
Answer: nausea, vomiting, abdominal distention, dec stool
Active infection is a contraindication for what?
Answer: elective surgery
What are some classic signs of DVT?
Answer: unilateral leg edema, local warmth, erythema, low grade fever (PS a negative
Homan’s is normal)
LITHIUM- bipolar range?
Answer: 0.6-1.2
What levels of lithium are considered toxic?
Answer: over 1.5

While taking lithium clients should:
Answer: • drink 1-2 L of fluid daily
• maintain normal sodium levels
• have their blood drawn regularly for therapeutic levels
What can cause lithium toxicity in a patient taking lithium?
Answer: Dehydration, decreased renal function, a diet low in sodium, and drug-drug
interactions, such as NSAIDs.
At what age does an infant have equal head and chest circumference?
Answer: 1 year
Normal weight gain in first 6 months in infants
Answer: 5-7 oz (140-196 g) a week
Normal weight gain from age 6-12 months
Answer: 3-5 oz (84-140 g) a week
At 12 months, an infants weight should be
Answer: tripled the birth weight
At 12 months can in infant go from standing to siting
Answer: yes
When does teeth eruption happen in infants?
Answer: 6-10 months
How to calculate the # of teeth a child should have in the first 24 months?
Answer: age of child in months minus 6
What are vegans at risk for since they don't eat animal products?
Answer: Tingling and numbness from vitamin B12 deficiency.

Vitamin B12 deficiency affects the entire nervous system from peripheral nerves to spinal cord
and brain
Answer: Vitamin B12 deficiency impacts the entire nervous system, affecting peripheral
nerves, the spinal cord, and the brain.
When do you not give a vaginal exam for a pregnant woman?
Answer: bleeding with unknown origin
What is suspected in a pregnant woman having painless vaginal bleeding after 20 weeks
gestation?
Answer: placenta previa
A tuft of hair at base of spine in a newborn indicates what?
Answer: spina bifida
Taking folic acid during pregnancy prevents what?
Answer: neural tube defects and spina bifida
UAPs CAN perform oral suction for pts during oral care
Answer: Unlicensed Assistive Personnel (UAPs) can perform oral suction for patients during
oral care, provided they are trained and supervised by a licensed nurse.
Normal albumin level
Answer: 3.5-5.0
What is the function of albumin?
Answer: to keep fluid from leaking out of vessels
When serum albumin is low this can cause
Answer: pitting edema
While talking with a delusional pt, focus on the pts feelings and reinforce reality
Answer: When interacting with a delusional patient, validate their feelings while gently
redirecting the conversation to reality to provide support without reinforcing the delusion.

For priority questions, anything with a new onset problem is usually first
Answer: In priority questions, new onset problems typically take precedence as they may
indicate an urgent or changing condition that requires immediate attention.
In therapeutic communication, don’t choose "why" questions
Answer: In therapeutic communication, avoid "why" questions as they can sound
confrontational and may put the patient on the defensive.
Birth weight should double at 6 month and triple at 12 month
Answer: Birth weight should double by 6 months and triple by 12 months.
When does the anterior fontanel close?
Answer: 18 month
When does the posterior fontanel close?
Answer: 2 months
What is typically seen in a retinoblastoma?
Answer: absence of red reflex. the pupil reflects a white color
What route is the best way to give burn victim meds?
Answer: IV
14 gauge catheter
Answer: fluids and drugs in prehospital or emergency setting or hypovolemic shock
18 gauge
Answer: infusing blood or large amounts of fluids in adults
Drug for fibromyalgia
Answer: • duloxetine (Cymbalta)
• it has both antidepressant and pain reliving affects
• it’s also prescribed for MDD

What is fibromyalgia?
Answer: abnormal CNS pain transmission and processing
Incentive spirometer use
Answer: piston rises with each deep inhalation
Why is it used?
Answer: to prevent atelectasis in post op clients
How many breaths are recommended?
Answer: 5-10 breaths per session every hour while awake
Process for breathing from an incentive spiromeyer
Answer: • sit in high Folwer’s or sitting
• seal lips tightly to prevent air leakage
• inhale deeply
• hold breath for 2-3 seconds
• exhale slowly
• breathe normal for several breaths before repeating the process
• cough at the end of the session
What is a serious side effect of statin drugs?
Answer: muscle aches and weakness
If myopathy is present, what will be very elevated?
Answer: creatinine kinase
Side effect of calcium channel blockers?
Answer: headaches
The use of radioactive iodine is contraindicated in
Answer: a pregnant client. all females should be tested before

Primary use of radioactive iodine?
Answer: • primary treatment for nonpregnant adults with hyperthyroid disorders (eg Graves
disease)
• RAI damages or destroys thyroid tissue and can result in hypothyroidism - clients need to
take thyroid supplementation (levothyroxine) for life
Lyme disease
Answer: initial symptoms: flu like ha, fever, myalgia, fatigue, bulls eye rash. give antibiotics
to prevent it from spreading to heart brain or joints
Any condition that causes a decrease in RR is at risk for developing
Answer: respiratory acidosis
Respiratory acidosis
Answer: low pH and high PACÓ
Metabolic acidosis
Answer: low pH and low HCǑ
Respiratory alkalosis
Answer: high pH and low PACÓ
Metabolic alkalosis
Answer: high pH and low HCǑ
Hypo ventilating can cause a buildup of carbon dioxide in the blood and can lead to
Answer: respiratory acidosis
Delusions of reference
Answer: CTS believe songs newspaper articles the radio and other things are personal and
significant to them
Somatic delusion
Answer: of having some physical defect or disease

Persecutory (paranoid) delusion
Answer: the belief of being treated in a malicious way
What is creatinine clearance
Answer: a measure of glomerular function and an indicator for renal disease progression it is
a 24 hour urine collection. the test must be started again if all urine in the 24 hours period is
not collected when the test begins, the first urine is taken and thrown away
What is palliative care?
Answer: manages symptoms, provide psychical support, coordinate care to relieve suffering
and improve quality of life for CTS with serious illnesses
Classic symptoms of a rotator cuff injury
Answer: shoulder pain & weakness, sever pain on arm abduction
What is a saline lock?
Answer: the cap that goes onto an IV insertion catheter. it allows you the ability to stop
giving iv fluids and then start again at a later time without having to do another venipuncture
Patient with the flu precautions
Answer: • droplet
• be in 3 feet when coughing or sneezing
• place mask on pt when transporting
What is the purpose of un-fractioned heparin?
Answer: to slow the time it takes the blood to clot thereby keeping the current clot from
getting bigger and preventing new ones from forming
Mono is caused by the
Answer: Epstein-barr virus
Is aspirin contraindicated in children and why?
Answer: yes because they can get Reye’s syndrome

Symptoms of mono?
Answer: • fatigue, fever, sore throat, splenomegaly, hepatomegaly, swollen lymph nodes
• contact sports should be avoided
What is normal CVP?
Answer: 2-8
What is CVP and what does it monitor?
Answer: • central venous pressure measuring right ventricular preload and reflects fluid
• volume status
• connect it to the proximal port to pressure monitoring system
How to assist a falling client?
Answer: stand behind and let out leg for supporting them to ground
Paraesthesia is an early sign of what?
Answer: neurovascular impairment
What is a major adverse effect of TNF inhibitors like infliximab or adalimumab?
Answer: infection because they cause immune suppression
positions
Answer: Sit in high Fowler's position or upright to optimize lung expansion.
For paracentesis
Answer: high fowlers
Enema administration
Answer: sims
After a lumbar puncture
Answer: supine

After liver biopsy
Answer: right side (to put pressure on the liver)
After cardiac catheterization
Answer: lie flat
Normal newborn RR
Answer: 30-60
Glucose levels are normally decreased 1 hour after birth
Answer: Glucose levels typically decrease within the first hour after birth due to metabolic
adjustments in the newborn.
What will damage to the occipital lobe cause?
Answer: visual disturbances
What do advanced care documents include?
Answer: health care proxy and living will
If pt comes into ER and says they have the worst HA of their life what could it be
Answer: an aneurysm
What is the most important med to give for anaphylactic shock?
Answer: IM epinephrine
What are Mongolian spots?
Answer: benign discoloration bluish Gray but usually fades
After an amputation, should the residual limb be elevated?
Answer: NO, to prevent hip flexion contractures
Buck Traction
Answer: immobilizes hip and femur fractures . the weight pulls the limb into traction. the
foot of the bed is elevated

Following total hip replacement, how is hip dislocation prevented?
Answer: • using an abductor pillow to maintain hip in a straight position.
• Don’t bend hip more than 90 degree
Before starting broad spectrum antibiotics, what must be done?
Answer: blood culture and sensitivity
What is HELLP related to pregnancy?
Answer: • Haemolysis
• Elevated Liver enzymes
• Low Platelet
• it is a severe sign of preeclampsia. symptoms may be RUQ pain nausea/vomiting and
malaise
Normal sodium
Answer: 135-145
Homonymous hemianopsia
Answer: loss in half of the visual field on the same side
Oxytocin
Answer: • induction or augmentation of labor
• should be given by an electronic infusion pump
• requires frequent mother/fetal assessment
• adverse effects: maternal hypotension and water intoxication
What are some seizure triggers?
Answer: excess caffeine, alcohol, sleep deprivation, and stress
Phenytoin
Answer: gingival hyperplasia
Don’t give NSAIDs with nasal polyps

Answer: NSAIDs should be avoided in patients with nasal polyps due to the risk of
exacerbating symptoms.
Don’t give beta blockers to asthma patient
Answer: Beta blockers should not be given to asthma patients, as they can provoke
bronchospasm.
Treat asthma with
Answer: Albuterol. Side effects: nausea/vomiting, insomnia, palpitations, tremor.
Proton pump inhibitors can increase the risk of
Answer: C-diff
Common proton pump inhibitors PPIs
Answer: Prazoles: omeprazole, lansoprazole, etc
Heparin is measured thru what levels?
Answer: PTT
Normal WBCs
Answer: 4,000-11,000
A patient on heparin should have aPTT of 1.5-2 times the normal range, which is 25-35
seconds.
Answer: A patient on heparin should have aPTT levels of 37.5-70 seconds, indicating
effective anticoagulation.
What is the causative agent of fifth disease?
Answer: human parvovirus
Fifth disease
Answer: • "slapped face"
• distinct red rash on cheeks
• its on communicable before the onset of symptoms

• spreads via respiratory
PKU
Answer: • low phenylalanine diet
• special infant formula
• no meat eggs or dairy
Warning signs of cancer
Answer: Caution
What do rales (crackles) indicate?
Answer: fluid in the lungs
Proper nasal spray administration
Answer: • high fowlers and
• head bowed slightly forward
• occlude one nostril and put it in open one
• point tip toward side and away from center
• spray while inhaling deeply
Normal platelets
Answer: 150,000-400,000
How to close a Hemovac
Answer: compress to create negative suction pressure and apply plug before releasing the
pressure
Most important to monitor bulimic pts when?
Answer: It is most important to monitor bulimic patients during and after episodes of binging
and purging for electrolyte imbalances and cardiac complications.
cystic fibrosis
Answer: • chest physiotherapy- to remove
• secretions that cause ineffective airway clearance

• CF is genetic
• High fat high calorie diet
• liberal fluid intake
When would you see dry shiny hairless skin?
Answer: peripheral atherosclerotic arterial disease
Wilms tumor is on the
Answer: • kidney presents as abnormal bulging on one side of child's abdomen
• do not palpate abdomen
Process for a burn at home
Answer: • soak in cool water to stop
• burring process
• remove clothing around burn
• cover with a clean dry cloth
• don’t put any meds or anything on the wound
Post surgical pts are usually prescribed which prn med to help from the opioid drugs they
were on
Answer: naloxone (Narcan) but it wears off in 1-2 hours so be prepared to give more than
once monitor RR and give oxygen
Cushing’s triad
Answer: bradycardia, slowed RR, widened pulse pressure
What is the normal pulse rate for a 1 year old
Answer: 100-160
Hypokalaemia can cause
Answer: heart arrythmias and muscle cramps and weakness
What do you closely monitor when giving furosemide?
Answer: • blood pressure

• potassium
• bun
• creatinine
What type of medication is clopidogrel?
Answer: antiplatelet
"Look, listen, feel" approach
Answer: The "look, listen, feel" approach is a method used to assess breathing by observing
the patient's effort, listening for sounds, and feeling for airflow.
PPL with long bone or pelvic fractures are at risk for developing what?
Answer: fat embolism
What patients are considered immunosuppressed?
Answer: on steroids/chemo, HIV positive, new post op, multiple chronic co morbidities,
splenectomy, diabetes, very young/elderly
What kind of precautions do you need for open lesions from shingles?
Answer: both contact and airborne precautions and a negative airflow room
Symptom of macular degeneration
Answer: loss of central field of vision
Seeing small flashes of light is a sign of
Answer: retinal detachment
What is presbyopia?
Answer: inability to see things close up
Some of the "CIN" antibiotics can cause what?
Answer: prolonged QT interval and can also cause hepatotoxicity so make sure to check liver
function tests

Diet for chronic kidney disease
Answer: restrict fluids, sodium, potassium, and phosphorus
What will you hear in lung during atelectasis?
Answer: fine crackles (high pitched popping sound)
Calcium channel blockers end in
Answer: "pine"
What are the most important adverse effects of calcium channel blockers?
Answer: dizziness, flushing, HA, peripheral edema, and constipation
Poor thiamine consumption can lead to Wernicke encephalopathy.
Answer: Poor thiamine consumption can lead to Wernicke encephalopathy. This condition is
characterized by confusion, ataxia, and ophthalmoplegia. If left untreated, it can progress to
Korsakoff syndrome, resulting in severe memory impairment.
Tracheostomy allow for 1 finger to fit under the ties
Answer: A tracheostomy should allow for one finger to fit comfortably under the ties to
ensure it is not too tight, allowing for proper airflow and avoiding pressure on the skin.
Main care with someone with a new track
Answer: prevent dislodgement
What is Docusate
Answer: stool softener
What are foods high in vitamin k?
Answer: leafy green veggies, asparagus, broccoli, kale, Brussel sprout, spinach
Some symptoms of tardive dyskinesia
Answer: unusually movements of arms, legs, face, body
Metoclopramide

Answer: can cause tardive dyskinesia
Therapeutic trough levels of vancomycin
Answer: 10-20
Normal creatinine level
Answer: 0.6-1.3
Normal BUN level
Answer: 6-20
Elevated bun and creatinine may indicate what?
Answer: nephrotoxicity
First level priority problems
Answer: • airway
• breathing
• circulation & cardiac (become first priority in cardiac arrest)
• vital signs
Second level priority problems
Answer: • altered mental status
• acute pain
• untreated medical problems (like hyperglycemia in client with diabetes)
• chronic pain
• acute elimination problems
• abnormal lab results
• risk for infection, safety
Where should the nurse listen for a bruit in abdominal aneurysm
Answer: epigastric/periumbilical area
What are TORCH infections
Answer: cause fetal abnormalities in pregnant women

What does TORCH stand for?
Answer: • T: toxoplasmosis
• O: other parvo- b 19, varicella zoster
• R: rubella
• C: cytomegalovirus
• H: herpes
How is toxoplasmosis acquired ?
Answer: exposure to infected cat feces or ingestion of undercooked meat or soil
contaminated fruits/vegetables
Patient with measles precautions
Answer: • airborne and negative
• pressure room
• N95 respirators
Priority intervention for eczema
Answer: prevent scratching
When are pancreatic supplements taken?
Answer: with or just before every meal
What will chronic pancreatitis show in stool
Answer: greasy, foamy, foul-smelling, fatty
How about biliary obstruction
Answer: light gray clay colored
RA causes bilateral symmetrical pain in small joints of hands wrists and feet
Answer: Rheumatoid arthritis (RA) causes bilateral symmetrical pain and swelling in the
small joints of the hands, wrists, and feet. This condition often leads to morning stiffness that
improves throughout the day.

Displacement
Answer: angry at boss and comes home yelling at wife
Warfarin
Answer: monthly INR
Spironolactone
Answer: potassium sparing diuretic
Liver dysfunction abnormalities
Answer: low albumin, high ammonia, elevated INR/PT, increased bilirubin, low platelets
symptoms of malignant hyperthermia
Answer: tachycardia, tachypnoea, rigid jaw, high fever. it is triggered by drugs used for
anaesthesia give them IV dantrolene
PTSD patients tend to
Answer: blame themselves. make sure to tell them it was not their fault
How to use the peak flow meter for asthma
Answer: exhale quickly and forcibly thru the mouthpiece
nursing care for acute pancreatitis
Answer: • NPO
• IV opioids
• IV fluids
• NG tube for suction
What puts a patient at risk for preterm labor?
Answer: infection like periodontal disease
Symptoms of active tuberculosis
Answer: • cough, fever/chills, malaise, weight loss, night sweats, anorexia, fatigue
• positive T skin test

• abnormal chest x-ray
• can transmit
Latent TB infection
Answer: positive TST, neg chest x-ray, asymptomatic, cant transmit
Steroids can convert latent to active TB
Answer: Steroids can weaken the immune system, potentially converting latent tuberculosis
(TB) into active TB. This risk underscores the importance of screening for TB before starting
steroid treatment.
What is an amniotomy?
Answer: artificial rupture of the membranes
What should the nurse do during an amniotomy?
Answer: Assess FHR before and after, note characteristics of amniotic fluid, assist client to
an upright position after the procedure
Symptoms of anaphylaxis
Answer: • oral/airway swelling
• dizziness
• tachycardic
• stridor/hoarseness
• wheezing
• rash
• pruritus
• flushing
• nausea/vomiting
• abdomen pain
Treatment of anaphylaxis
Answer: IM epinephrine
Before starting on statin drugs, what must be checked?

Answer: liver function tests because they can cause hepatotoxicity
What can statins cause?
Answer: muscle aches and can lead to rhabdomyolysis
Aphasia
Answer: inability to express thoughts and comprehend language
What does a pt with Marfan syndrome look like
Answer: tall and thin. they often have heart abnormalities so contact sports are discouraged
So no contact sports with Marfan syndrome
Answer: No contact sports are recommended for individuals with Marfan syndrome due to
the increased risk of cardiovascular complications.
What is the Bishop score?
Answer: assesses and rates cervical favourability and readiness for induction of labour. a
score above 8 is good
Carbidopa-levodopa
Answer: treats Parkinson's disease and improves spontaneous activity seen with this
condition.
What does a person see with glaucoma eye
Answer: tunnel vision
What does a sacral dimple on a newborn signify?
Answer: spina bifida
What is a normal newborn HR
Answer: 110-160
What type of drug is clopidogrel, prasugrel, and ticagrelor
Answer: antiplatelet

Prior to an EEG which diagnoses seizures:
Answer: • non caffeine, stimulants or depressants
• wash hair
What is Duchenne muscular dystrophy?
Answer • childhood form of muscular dystrophy
• X-linked recessive
• lacks a protein called dystrophin for proper muscle stabilization
• muscles of proximal lower extremities are affected first
• they exhibit the Gower sign to help them stand
• remove clutter to prevent falls
What is the Gower sign?
Answer: uses ones hands to rise from squat to a chair
X linked recessive
Answer: carried by females and affects males
What is methotrexate?
Answer: • antineoplastic, immunosuppressant to treat malignancies
• chemotherapy, rheumatoid arthritis, psoriasis, and for abortions
What are the major side effects with methotrexate?
Answer: bone marrow suppression, hepatotoxicity, fetal death. you don’t need monthly eye
exams when taking it
Remember for lung positioning:
Answer: Good Lung Down
Transport techs need to/don’t need to know which info
Answer: • need: pertinent info to transport patient
• don’t need: exact diagnosis

You can put last names on boards in nurses station
Answer: Last names can be displayed on boards in the nurses' station to maintain patient
confidentiality while facilitating communication among staff.
What is the only medication approved for ALS?
Answer: Riluzole (Rilutek)
Pneumonia vaccine
Answer: 65 and above need 2 vaccines
Adverse reactions to gentamicin
Answer: ototoxicity and nephrotoxicity
What is an expected side effect symptom of nitro-glycerine?
Answer: headache
What do some calcium channel blockers, like nifedipine, cause?
Answer: edema from vasodilation
Calcium channel blockers end in
Answer: PINE = vasodilation
Radioactive iodine (RAI)
Answer: • treats hyperthyroidism destroys thymus gland
• take precautions to not spread this to others. aka everything private
Myasthenia gravis clinical manifestations
Answer: • ocular (ptosis) & facial muscle weakness - difficulty speaking, swallowing,
chewing
• can progress to respiratory muscles and affect breathing
1st line treatment for Myasthenia Gravis
Answer: Pyridostigmine

A chest tube and catheter should both be placed
Answer: below chest and bladder
What kind of environment do autistic kids need?
Answer: a private calming one with minimal stimulation
Rifampin
Answer: turns liquids red
Signs of developmental dysplasia of the hip?
Answer: • thigh folds
• laxity of the hip joint
Hypoglycemia symptoms
Answer: tremors, palpitations, anxiety, restlessness, sweating, and pallor
What type of drug is celecoxib?
Answer: NSAID
If patient has an infection, do not do total anything replacement
Answer: If a patient has an infection, do not perform total joint replacement surgery.
Classic symptoms of neurogenic shock
Answer: hypotension, bradycardia, and pink an dry skin
What should you do for any kind of shock?
Answer: Administer fluids.
What number is considered sever neutropenia?
Answer: less than 500
Aura comes before what?
Answer: seizure

Back pain after catheterization could be what?
Answer: retroperitoneal bleeding
Parkland formula... what is the formula and actually how much is given?
Answer: • 4 × weight in kg × % of body burned
• half of the calculated fluid must be given in the first 8 hours
Low platelets = ICP bleeding = change in LOC
Answer: Low platelets can lead to intracranial bleeding, resulting in changes in level of
consciousness (LOC).
Kidney biopsy
Answer: Bleeding
Azathioprine
Answer: immunosuppressant drug causing bone marrow suppression and increases risk for
infection
When is digoxin held in kids?
Answer: pulse less than 90
ARBs and ACE inhibitors are contraindicated in
Answer: pregnancy
What does elevated CVP mean
Answer: right vent failure or fluid overload
C diff room needs
Answer: single
Rubella titter is contraindicated in
Answer: pregnancy
Mass casualty goal

Answer: greatest good for greatest # of ppl
Furosemide can cause what
Answer: ototoxicity
Lithium toxicity
Answer: GI side effects mainly
Client self-determination act
Answer: clients must be given info about their rights to identify written directions about the
care that they wish to get in the event they become incapacitated and are unable to make
healthcare decisions. Asked about advance directive on admission
2 types of advance directives
Answer: Two types of advance directives are living wills and durable power of attorney for
healthcare. Living wills outline specific medical treatments a person wants or doesn’t want,
while durable power of attorney designates someone to make healthcare decisions on their
behalf.
Instructional directive
Answer: lists medical treatment the client wants to omit or refuse if they become unable to
make decision
Durable power of attorney
Answer: • health care proxy
• chosen by client to make health care decision on clients behalf id client can no longer make
decisions
Informed consent is not needed for an unconscious adult comes into the er why?
Answer: Because delaying treatment could causes injury or death to the patient
Intractable diarrhoea
Answer: chronic, diarrhoea that is hard to control

Influenza is transmitted how?
Answer: droplet
What does a DNR exactly mean to withhold?
Answer: CPR and other lifesaving measures
What is Tofranil?
Answer: tricyclic antidepressant
What is osteoporosis
Answer: low bone density
Stage 1 ulcer
Answer: • intact skin
• non-blanchable with localized redness
Stage 2
Answer: • blister, abrasion, shallow open ulcer
• red pink wound with no sloughing
Stage 3
Answer: full thickness skin loss with visible sub q fat
Stage 4
Answer: • full thickness skin loss
• exposed muscle tendon or bone
Unstageable
Answer: • full thickness skin loss
• ulcer base covered by slough and eschar that needs to be removed to stage
Which long acting insulins can you not mix?
Answer: glargine and detemir

What does lead poisoning cause?
Answer: neurological and motor impairment
Strategies to prevent accidental drug overdoses in children
Answer: • keep medications out of sight in a locked drawer or cabinet
• put them away after each use
If a chest tube gets dislodged, immediately cover with something sterile or clean if you don’t
have sterile. what are we trying to prevent here?
Answer: air from getting into the pleural cavity
What is happening in ventricular trigeminy?
Answer: PVCs are occurring every third heartbeat
It’s dangerous when someone is throwing PVCs why?
Answer: they could go into a lethal dysrhythmia
What is the battle sign?
Answer: bruising behind the ear
What could this indicate?
Answer: skull fracture that could cause brain injury if not treated
Risperidone - what class and what is it used to treat?
Answer: antipsychotic used to treat schizophrenia and bipolar
Examples of extrapyramidal symptoms
Answer: • pseudo parkinsonism
• dystonia
Pseudo parkinsonism
Answer: A medication-induced temporary constellation of symptoms associated with
Parkinson's disease, (eg masklike face, shuffling gait, rigidity, resting tremor, psychomotor
retardation [bradykinesia])

Dystonia
Answer: abnormal muscle movements of the face, neck, and trunk caused by sustained
muscular contractions (eg torticollis, oculogyric crisis, opisthotonos)
What is akathisia?
Answer: restlessness with inability to sit still
Tardive dyskinesia
Answer: involuntary movements like face tongue trunk that you can’t control
Neuroleptic malignant syndrome
Answer: fever, muscle rigidity, altered mental status
Anticholinergic effects
Answer: dry mouth and constipation
What are some meds that treat EPS?
Answer: benzos like "pam" drugs and Benadryl
Blood products should not be left at room temp for more than 30 minutes before being given
Answer: Blood products should not be left at room temperature for more than 30 minutes
before administration.
Drugs contraindicated in patients with hyperkalaemia
Answer: 1. ACE inhibitors
2. Angiotensin 2 (ARBs)
3. Potassium-sparing diuretics
How often should ostomy bags be changed?
Answer: every 510 days
When should an ostomy bag be emptied? (when it is how full?)
Answer: when it is 1/3 full

What is preeclampsia?
Answer: • new onset of hypertension
• plus proteinuria after 20 weeks gestation
Signs of preeclampsia
Answer: • BP > 140/90
• proteinuria >300 mg of protein in urine in 24 hours
• headache
• visual changes
• 2+ pitting edema
Preeclampsia can lead to what severe thing?
Answer: Seizures
What is Amphotericin B?
Answer: antifungal med used to treat systemic fungal infections
It is commonly associated with what?
Answer: severe adverse reactions like hypotension, fever, chills, and nephrotoxicity
Ulcerative colitis diet
Answer: low residue, high protein high calorie
Kids with autism spectrum disorders are usually
Answer: visual Don’t touch them or make eye contact
Dopamine function
Answer: treat distributive shock and maintain cardiac output
1 kg is
Answer: 2.2 lbs
Albuterol is a

Answer: bronchodilator
Expected side effects of albuterol
Answer: tachycardia, palpitations, and tremors
What is obstructive sleep apnea?
Answer: intermittent airflow blockage during sleep
Sleep apnea features
Answer: • Nighttime:
• periods of apnea
• loud snoring
• interrupted sleep
Daytime
• morning headaches
• irritability
• excessive sleepiness
Other
• difficulty concentrating
• forgetfulness
• mood changes
• depression
Sleep apnea interventions
Answer: • continuous positive airway pressure device at night
• limiting alcohol intake at bedtime
• weight loss and exercise
• avoiding sedating medications (eg benzos, sedating antidepressants, antihistamines, opiates)
Stroke patients are at a high risk for
Answer: injury/falls because of one sided paralysis and confusion
Clients with ESRD are unable to excrete which electrolyte?
Answer: potassium; so they should eat a diet low in potassium

ESRD Potassium
Answer: In end-stage renal disease (ESRD), potassium levels are typically elevated
(hyperkalaemia) due to impaired kidney function.
What are some foods that are high in potassium?
Answer: bananas, strawberries, legumes, green leafy veggies, potatoes, carrots, tomatoes,
melons, avocados, whole grains, most fish, beef, pork
Accidental extubation is a medical emergency. if it happens, what does the nurse do?
Answer: immediately deliver rescue breaths using a bag valve mask and 100% oxygen until
they can be reintubated
If patients are having an ischemic stroke, they are given tissue plasminogen activator tPA to
dissolve the clot. how soon must tPA be given?
Answer: within 3-4.5 hours of onset of symptoms
Does TPA do?
Answer: dissolve all clots in the body
What are the contraindications for giving TPA?
Answer: • thrombocytopenia
• coagulation disorders
• major surgery within the last 14 days
Thrombocytopenia
Answer: >100,000 platelet count
When a patient is taking lithium, be careful when taking which medications in addition
because they can increase lithium levels and lead to toxicity?
Answer: thiazide diuretics, NSAIDS, and antidepressants
What should a patient know when taking lithium
Answer: • Sodium is really important

• get adequate sodium
• drink 2-3 L/day
Therapeutic level of lithium?
Answer: 0.6-1.2
Over what number is lithium considered toxic?
Answer: over 1.5
Lithium toxicity symptoms
Answer: • acute: GI disturbances:
• nausea, vomiting, diarrhoea
• neuro finding can occur later
• chronic- neurologic ataxia, confusion, tremors
Nitro-glycerine is why type of drug?
Answer: antianginal vasodilator that may cause a headache
What are wheezes?
Answer: continuous, high pitched musical adventitious sounds caused by air movement
through narrow bronchi and bronchioles
Placental abruption
Answer: the placenta prematurely detaches from the uterine wall
What are the priorities of care for suspected placental abruption?
Answer: Monitor the mother for fluid status and fetal heart tones because it can cause
maternal haemorrhage and/or interruption of fetal oxygen supply.
What are some symptoms of placental abruption?
Answer: frequent contractions, abdomen pain, dark red vaginal bleeding
Placenta previa
Answer: • when the placenta covers

• the opening of the mothers uterus
• it usually presents with painless vaginal bleeding
What does an ST elevation indicate?
Answer: myocardial infarction because one or more of the coronary arteries that supply
blood to the myocardium are occluded
What are some causes of the low pressure limit on a ventilator?
Answer: loss of airway (extubation), endotracheal or tracheostomy cuff leak, or tubing
disconnect, all of which impair airway and ventilation
What number is the normal, expected deep tendon reflex?
Answer: 2+
Presence of neurological change in pregnant women may indicate what?
Answer: worsening preeclampsia
What is clonus?
Answer: cause is neurological; it is rhythmic muscle spasms
How to assess for clonus?
Answer: nurse dorsiflexes the foot with one hand and supports the leg/ankle with the other.
when positive clonus is shown, rhythmic jerking beats of the foot are present as the foot is
released
Reactive nonstress test vs nonreactive nonstress test
Answer: reactive nonstress test you have a baseline of 110-160/min, moderate variability and
2 or more accelerations in 20 mins and nonreactive does not meet criteria for reactivity
Is reactive nonstress test positive or negative?
Answer: Positive
Where to look for certain things in dark skinned patients

Answer: In dark-skinned patients, assess for certain signs in areas with less pigmentation,
such as the palms, soles, or mucous membranes.
Jaundice?
Answer: palms of hands and soles of feet
Petechiae?
Answer: buccal mucosae and conjunctivae of the eyes
Cyanosis?
Answer: nail beds of fingers/toes
What are the signs and symptoms of a myocardial infarction?
Answer: Chest pain, diaphoresis, dyspnea, and anxiety.
TORCH infections cause what?
Answer: fetal abnormalities
Late or variable deceleration indicate what in pregnancy?
Answer: not good and need further teaching
Nursing interventions that decrease ICP?
Answer: • HOB at 30 degrees
• keep head and body midline
• give stool softeners
• suction only when necessary
• provide a calm environment with dimmed lights
• treat fever
Symptoms of RSV in infants
Answer: excessive mucous production, rhinorrhoea, fever, cough, lethargy, irritability, poor
feeding
RSV interventions

Answer: • give oxygen
• elevate HOB
• airway suction
• give antipyretics
• IV fluids
• contact and droplet isolation
Bipolar patient comes in and is experiencing acute mania. what is the priority?
Answer: Not risk prone behavior because physiological needs like food take priority
(remember maslows). so imbalanced nutrition would be the priority
Maslow's Hierarchy of Needs (Prioritization)
Answer: Greatest to Least Priority:
1. Physiological (survival) - O2, fluids, nutrition, body temp, elimination, sleep, shelter, sex
2. Safety & Security (physical, psychological) - protection, security
3. Love & Belonging - family, affection, friendships
4. Self-Esteem - achievement, confidence
5. Self-Actualization - fulfilment
Having a patient bring their meds to each appointment is the appropriate thing to do
Answer: Having a patient bring their medications to each appointment is important for
accurate medication reconciliation and assessment.
Botulism is what and caused from?
Answer: GI absorption of toxin produced by clostridium botulinum from improperly canned
or stored food. found in soil and can grow in food. look out for swollen/bulging cans
What does botulism cause?
Answer: muscle paralysis
Children under what age should not be given honey?
Answer: 1 year
Serious form of preeclampsia condition called HELLP stands for what?

Answer: Hemolysis, Elevated Liver enzymes, Low platelets- delivery right away
Swelling of the hands feet and face are expected findings of what pregnant condition?
Answer: preeclampsia
Preeclampsia 2 main things:
Answer: high blood pressure and protein in urine edema is expected but not part of the
criteria
Complications of preeclampsia?
Answer: eclampsia, placental abruption, and HELLP
How is hepatitis A transmitted?
Answer: fecal-oral route
How is hepatitis B transmitted?
Answer: • blood, semen, vaginal secretions
• "B" for body fluids
Early symptoms of hepatitis B?
Answer: nonspecific like malaise, nausea/vomiting, abdomen pain
Later signs of hepatitis B?
Answer: jaundice, weight loss, clay colored stools, thrombocytopenia
What are some symptoms that a pregnant woman has HELLP?
Answer: RUQ pain, nausea, vomiting, malaise
Delirium signs
Answer: sudden onset of fluctuating mental status changes and inattention with disorganized
thinking and/or altered level of consciousness. think of the patient that I had that was lying in
bed and reaching up grabbing something in the air with her hand that was not there
Delirium vs dementia

Answer: dementia has a slow onset usually with normal attention
Infertility
Answer: inability to conceive after unprotected intercourse for more than 12 months
Factors contributing to female infertility
Answer: 1. hormonal dysfunction (eg PCOS) with anovulatory cycles
2. High or low BMI (normal: 18.5-24.9 kg/m2)
3. Maternal age >35
4. Endometriosis
5. Recurrent infections (eg chlamydia, PID)
Once a woman's water breaks, this leaves her susceptible to what?
Answer: infection
If GBS status is unknown in a pregnant woman, always give what?
Answer: prophylactic antibiotics
When are pregnant clients tested for GBS?
Answer: 35-37 weeks
If a pregnant women is positive for GBS, what happens?
Answer: she gets prophylactic antibiotics so that she does not pass it to her baby during labor
because it can cause harmful things like pneumonia or neonatal GBS sepsis
Carbidopa-levodopa is the drug for what?
Answer: Parkinson's disease.
What dose specifically help with?
Answer: bradykinesia. can also improve tremor and rigidity
What is the role of the combo drug carbidopa-levodopa?
Answer: levodopa converts to dopamine so more is available and carbidopa helps prevent the
breakdown of levodopa

Once is something to remember about carbidopa-levodopa?
Answer: never stop it suddenly
Symptoms of PE
Answer: • SOB
• RR >20/min
Even after the diagnosis of a DVT, is walking suggested?
Answer: Yes
Where is a PICC line inserted into?
Answer: basilic or cephalic veins which go into the superior vena cava
During dressing changes of a PICC line, what should a nurse check?
Answer: the external length of the PICC
After abdominal surgery, do you expect to hear bowel sounds?
Answer: no because postop paralytic ileus
Isotretinoin is what?
Answer: Accutane (acne med)
What is the relationship between Accutane and pregnancy?
Answer: Do Not Take if pregnant or planning to become. can cause serious birth defects.
pregnancy category X
Females prescribed Accutane must have what
Answer: • 2 negative pregnancy tests before starting
• must use 2 forms of contraception
• enrol in a risk management program to make sure these are both being followed through
Loop diuretics like Furosemide can cause what?
Answer: hypokalaemia

What is an example of potassium sparing diuretics?
Answer: Spironolactone (Aldactone)
It is normal for children to regress while hospitalized
Answer: Yes, it is normal for children to experience regression during hospitalization due to
stress and changes in routine.
Signs of Epiglottitis
Answer: • sore throat, dysphagia, drooling, respiratory distress, and sitting up/leaning
forward
• An emergency- prepare for emergency airway
• Intubation/tracheostomy
Signs of croup
Answer: upper respiratory tract symptoms, hoarseness, barking cough, stridor, respiratory
distress
Urinary retention is an expected side effect of opioids
Answer: Yes, urinary retention is a common side effect of opioids.
Majority cases of epiglottitis are caused by what?
Answer: • HIB.
• if kids are vaccinated, this is rarely seen
Most cases of epiglottitis are prevented by what?
Answer: standard immunizations
With a balloon tamponade tube, what should always be kept at the bedside?
Answer: scissors
Haemophilia Patient Education
Answer: 1. Avoid medications that inhibit platelet aggregation (eg ibuprofen & aspirin)
2. Avoid IM injections (Sub Q is preferred)

3. Avoid contact sports & safety hazards (noncontact activities - swimming, jogging, tennis and use of protective equipment are preferred)
4. Dental hygiene is necessary - soft toothbrushes should be used
5. Medic Alert bracelets should be worn at all times
Most frequent sites of bleeding in a haemophiliac are where?
Answer: the joints
Contraindication meds for heart failure
Answer: NSAIDS
UAPS can assist with passive and active ROM exercises that they were taught by the nurse or
PT
Answer: Yes, UAPs can assist with both passive and active range of motion exercises as
instructed by the nurse or physical therapist.
Thiazolidinediones like rosiglitazone and pioglitazone treat type 2 diabetes. these drugs can
worsen what condition?
Answer: heart failure
When you think hyperkalaemia, think what?
Answer: Cardiac Problems
What can cause pseudo hyperkalaemia?
Answer: haemolysis or clotting during the blood draw
Hyperkalaemia priority interventions
Answer: 1. Administer IV 50% dextrose + regular insulin rapidly
2. If ECG changes are present, give calcium gluconate FIRST
Normal labor symptoms
Answer: intermittent pain with contractions, small amount of blood-tinged mucus "bloody
show"

Placental abruption symptoms
Answer: sudden-onset vaginal bleeding, abdominal pain, frequent uterine contractions
Placenta previa symptoms
Answer: painless vaginal bleeding, ultrasound shows placenta covering cervical OS
After a med error happens, what should you always do first?
Answer: assess the patient
"When on crutches, look forward when walking and not down at your feet."
Answer: When using crutches, maintaining a forward gaze improves balance and safety.
What ABG values indicate ARF?
Answer: • low PaO2 and
• high PaCO2
How to fix ARF immediately?
Answer: high concentrations of O2
Always try least restrictive before using restraint. NCLEX wants you to know this so most
likely the answer is not going to be apply restraint.
Answer: Always attempt least restrictive measures before considering restraints, as
emphasized by NCLEX guidelines.
Remember the 5Ps to compartment syndrome
Answer: paresthesias, pain, pressure, pallor, pulselessness
What can cause atelectasis after surgery?
Answer: retained secretions
A patient with a total knee arthroplasty should be what by discharge?
Answer: fully weight bearing
Left side heart failure symptoms

Answer: pulmonary edema, dyspnoea, orthopnoea, crackles
Right side heart failure
Answer: JVD, hepatomegaly, splenomegaly, ascites, edema
What does the peak mean when giving insulin?
Answer: when the insulin works to its maximum to lower the blood sugar
Rapid acting insulin peak (as part, lispro, glulisine)
Answer: 30mins-3 hours
Short acting (regular) insulin peak
Answer: 2-5 hrs
Intermediate acting (NPH)
Answer: in 4 hours
Long acting (detemir, glargine)
Answer: 4-9 hrs
Memorize that insulin paper I have
Answer: Clarify key points about insulin if you had like.
Always report bright red blood to hcp, dark blood is ok
Answer: Yes, always report bright red blood to the healthcare provider, as it may indicate
active bleeding, while dark blood can be less concerning.
How to give morphine iv push
Answer: dilute and give slowly over 3-5 mins
Abdominal surgery and other procedures with bowel manipulation case what for the bowels
for the first 24-48 hours?
Answer: absent bowel sounds

How are bowel sounds determined absent
Answer: nurse listens for 2-5 mins in each quadrant
Differences between SIRS, septic shock, and mods
Answer: SIRS is a systemic inflammatory response, septic shock is severe sepsis with
persistent hypotension, and MODS is the failure of multiple organ systems often following
sepsis.
Order of progression from sirs to mods
Answer: SIRS → sepsis → septic shock → MODS
SIRS
Answer: generalized inflammatory response to an infectious or non-infectious insult to the
body (fever, tachycardia, tachypnea)."
Sepsis
Answer: presence of infection along with systemic manifestations of infection
Septic shock
Answer: sepsis induced hypotension despite adequate fluid resuscitation
Desired MAP level in Septic or Anaphylactic Shock
Answer: >65 mm Hg
MODS
Answer: septic shock plus multiple organ system damage (2 or more organs)
What is the ABG most consistent with diabetic ketoacidosis?
Answer: metabolic acidosis
Diabetic ketoacidosis

Answer: severe form of hyperglycemia, >250. glucose can’t be taken out of the bloodstream
without insulin which they can’t produce. the body begins to break down fat stores into
ketones
What is one golden sign of diabetic ketoacidosis related to breath?
Answer: Kussmaul respirations (fruity smell)
ADHD kid acting out of control
Answer: have them deep breath and calm down
Key thing to remember about delirium is that it is ACUTE
Answer: Yes, a key aspect of delirium is that it is an acute and sudden change in mental
status.
Delirium
Answer: acute, consciousness impaired, fluctuating, reversible
Dementia
Answer: onset is gradual (months to years), consciousness intact, progressive, irreversible
Quality improvements should be objective and measurable
Answer: Quality improvements should be objective and measurable to ensure effectiveness
and accountability.
MMR is a live vaccine and live vaccines are contraindicated during what?
Answer: pregnancy
Some examples of live vaccines are:
Answer: MMR and varicella
when is the best time to give a pregnant client that is nonimmune to MMR the vaccine?
Answer: immediately postpartum
Acute pericarditis assessment

Answer: • sharp pleuritic chest pain
• aggravated on inspiration and coughing
• assessment shows pericardial friction rub
How to relieve pain in acute pericarditis?
Answer: sit up & lean forward
Acute pericarditis treatment
Answer: NSAIDs and colchicine (anti-inflammatory)
Pleurisy
Answer: • manifests with pleural friction rub - loud, rough rubbing or grating sound heard
throughout inspiration & expiration that's caused by the pleural surfaces rubbing together
Crackles - when are they heard?
Answer: heard typically only during inspiration
How should a person with COPD breathe?
Answer: pursed lip breathing
Pursed lip breathing instruction
Answer: - Regular practice (eg 5-10 min 4 times a day)
1. relax neck & shoulders
2. inhale 2 seconds through nose while mouth closed
3. exhale 4 seconds (or twice as long as inhalation) through pursed lips.
What does enteral mean?
Answer: given through the GI tract (orally or feeding tube)
Parenteral?
Answer: IV route
Is a central venous catheter CVC?
Answer: put in a "central" vein (subclavian, internal jugular, femoral)

Where is gluten found?
Answer: Brow- barley, rye, oats, wheat
Important education for celiac disease
Answer: gluten-free diet for the rest of their lives. why? gluten damages intestines (small)
that’s needed to function properly for nutrient absorption rice, corn, and potatoes are allowed
because they are gluten free read food labels of all processed foods because some may
contain hidden gluten
What is the first sign of puberty change in boys?
Answer: testicular enlargement
Unfractionated heparin, what does this mean?
Answer: Unfractionated heparin is a form of heparin that has a variable molecular weight
and is used for anticoagulation.
Orthostatic hypertension is a common side effect of most of what type of meds?
Answer: antihypertensives
Statins can cause which adverse effect?
Answer: muscle aches
When can fetal sex be determined?
Answer: end of 12 weeks
What is polycythaemia?
Answer: too many RBCs
Target therapy for polycythaemia
Answer: phlebotomy to remove some of those RBCs
What is monitored carefully after an abdominal aortic aneurysm (AAA) repair ?

Answer: 1. Pulses (femoral, posterior tibial, dorsalis pedis), skin color , and temperature of
lower extremities 2. renal status so things like BUN, creatinine, and urinary output
Pulse assessment post abdominal aortic aneurysm (AAA) repair
Answer: • Pulses can be absent for 4-12 hours after surgery due to vasospasm
• pedal pulse decreased from baseline or absent pulse with a painful, cool, or mottled
extremity can indicate presence of arterial or graft occlusion (life-threatening)
Skin cancer mnemonic ABCDE what does it stand for?
Answer: • Asymmetry
• Border irregularity
• Color changes
• Diameter more than a pencil eraser
• Evolving or changing in any way
Milking chest tubes
Answer: generally contraindicated because it messes with the pressure changes in the pleural
space
If a patient has a chest tube and all of the sudden the drainage stops, what are some
interventions?
Answer: • lung sounds (because maybe the lungs have re expanded and there is no more
drainage)
• reposition patient
• get them to cough and deep breathe
What volume of chest tube drainage should be reported to HCP
Answer: >100 mL/hr
Care of anorexic patient
Answer: • determine patient daily calorie requirement
• monitor weight at same time every day with same scale, same amount of clothes on
• limit exercise initially because they are obsessed
• reflecting with the patient on triggers

• don’t encourage a log of food because this contributes to patient's obsession with food
After a thyroidectomy, there should not be swelling of the neck or increased pain. what could
this indicate?
Answer: hematoma formation or increased tissue inflammation which affect airway patency.
this patient should be seen first
Hepatic encephalopathy symptoms
Answer: • complication of ESRD
• build up ammonia in the blood
• lethargy, confusion, slurred speech, can lead to coma watch for asterixis- flapping hand
tremors
What med is given for encephalopathy?
Answer: Lactulose - decreases intestinal ammonia absorption (gets rid of the ammonia)
Stress ulcers can develop in critically ill patients so what helps with this?
Answer: enteral feedings
Asthma
Answer: frequent cough at night
How long should an acceleration last on a fetal heart strip?
Answer: 15 min above the baseline lasting for at least 15 seconds means fetal movement
Is moderate variability consider normal?
Answer: yes and is 6-25/min
Late or variable decelerations are BAD- inadequate fetal oxygenation
Answer: Late or variable decelerations indicate inadequate fetal oxygenation and can be a
sign of distress.
What is the Allen's test?

Answer: determines patency of the ulnar artery. patient makes a fist and radial and ulnar
arteries are occluded. patient opens fist and pallor is shown on hand from non blood flow.
nurse releases pressure on arteries and blood flow is returned to hand. Allen’s test should be
positive and if negative, another artery should be used
Before ABGs are taken, what test must be performed?
Answer: the Allen's test
Can UAPs transfer a dead client to the morgue?
Answer: yes
Cardinal physical sign of Parvo aka fifth disease is what?
Answer: slapped cheek rash. spread person to person especially with respiratory secretions
Remember with bulimia, which gland is swollen?
Answer: parotid gland
What types of drugs are used to treat ADHD?
Answer: stimulants like Ritalin and Adderall
What is the trade NAE for methylphenidate?
Answer: Ritalin
What are some side effects of stimulants?
Answer: • decreased appetite and weight loss
• hypertension and tachycardia
• restlessness or insomnia
• vocal/motor tics
• addiction/misuse potential
If on oxytocin for induced labor and having more than 5 contractions in 10 minutes what
should the nurse do?
Answer: stop the infusion

What to do if patient is on oxytocin and late decals, fetal bradycardia, tachycardia, and
decreased variability happen, what does the nurse do in order?
Answer: • stop the oxytocin right away
• put patient in side lying position
• give oxygen (10L via face mask)
• IV fluid bolus
• maybe give terbutaline
• notify HCP
• document
Things that UAP are allowed to do
Answer: 1. passive/active ROM exercises
2. transport body to the morgue
3. pick up something from pharmacy
4. Measurement of vital signs
5. Fingerstick glucose testing
6. Personal hygiene & skin care
7. Oral care
8. Measurement of urine output
Burning at the IV site can indicate and lead to what?
Answer: can indicate phlebitis which can lead to thrombophlebitis (blood clot)
PCA pumps have a running IV line attached to deliver a flush each time a dose of the PCA is
delivered, this is to make sure the patient gets all of the pain med and thus has adequate pain
control
Answer: PCA pumps have a running IV flush to ensure complete delivery of pain medication
and maintain adequate pain control.
when is the only time that procedures can be performed without getting informed consent
first?
Answer: lifesaving procedures
If a patient has phantom limb pain after an amputation, what can the nurse do for comfort?

Answer: wrap and apply ice/heat
Active infection is a contraindication for elective surgeries
Answer: Active infection is a contraindication for elective surgeries due to increased risks of
complications.
What are signs of a bowel obstruction?
Answer: nausea, vomiting, abdominal distention, and dec stool production
Diverticular disease causes what?
Answer: protrusions form off the large intestine- they can become inflamed and infected
causing acute diverticulitis
What is the care for acute diverticulitis?
Answer: • let the bowel rest and let the inflammation go down
• IV antibiotics since bacteria may contribute to a flare up
• NPO
• IV fluids to prevent dehydration
• bed rest
• possible NG suction
• any procedure that causes abdomen pressure or that increases peristalsis could perforate or
rupture the diverticula and should be avoided
Metronidazole (Flagyl)
Answer: an antibiotic
Where in the chest tube drainage system is gentle, continuous bubbling normal?
Answer: suction control chamber. if more water is needed, add sterile water
Where should there NOT be continuous bubbling in the chest tube drainage system?
Answer: in the air leak monitor
What is the purpose of the water seal chamber in a chest tube

Answer: to prevent air from flowing to the client. up and down movement here with
inspiration and expiration is normal. as the lung re expands this will decrease
Should Not Be Continuous Bubbling In The Air Leak Monitor
Answer: Continuous bubbling in the air leak monitor indicates a possible air leak and should
be addressed promptly.
What are preventatives given before and after surgery to prevent venous thromboembolism?
Answer: heparin, enoxaparin, love NOx
_______ before and after surgery to prevent infection?
Answer: Ancef
meds common for an open fracture?
Answer: • Cefazolin
• (Ancef), tetanus toxoid, Toradol, opioids
What is Toradol?
Answer: NSAID used for inflammation and pain
What is Ancef?
Answer: cephalosporin antibiotic that’s active against skin flora like staff, given before and
after surgery
RSV/bronchiolitis causes what?
Answer: Respiratory secretions. Use saline nose drops and then suction the nares to remove
secretions. Give fluids.
Bronchiolitis
Answer: begins with upper respiratory symptoms like rhinorrhoea/congestion and then goes
to lower respiratory symptoms like tachypnoea, cough and wheezing
If patient has pertussis’s infection, family members should be treated

Answer: Yes, family members should be treated for pertussis to prevent transmission and
ensure protection.
It is normal for some children to have a reaction to MMRV vaccine 5-12 days after getting it
symptoms like
Answer: low grade fever, mild rash, swelling/redness at injection site, irritable, restless.
watch that their temp does not get too high because this could lead to febrile seizures. tell
them to give acetaminophen if temp over 102
Giving aspirin to children can cause what
Answer: Reyes syndrome
Acute lithium toxicity presents early with what and later with what
Answer: • early/acute: GI disturbances
•later/chronic: neurologic like ataxia, confusion, tremor, or diabetes insipidus manifestations
like polyuria, polydipsia
What is tiotropium (Spiriva) and what/how is it used for?
Answer: long acting, 24 hour, anticholinergic inhaled med used to control COPD. given
using capsule-inhaler system where the capsule is placed in the device and a bottom is
pressed to put a hole in the capsule and the powder is dispersed thru the inhaler. it should not
be used as a rescue med for example if someone has trouble breathing, that’s where you
would use the short acting like albuterol
Important teaching for tiotropium
Answer: do not take the capsule orally even though it comes in a capsule and look like you
do. The bottom must also be pushed on the inhaler for the med to be dispersed
Ipratropium
Answer: is the short acting anticholinergic used as a rescue med for COPD and asthma
Is a child is having separation anxiety in the hospital, what are some things nurses can do to
reduce the anxiety

Answer: have parent leave familiar objects in room, do not leave them alone when upset,
establish daily schedule that’s familiar to the one at home, give child time to play
Many people use St. John's wort to treat what?
Answer: depression
What meds does St. John's wort interact with
Answer: antidepressants like SSRIs, SNRIs, MAOIs can increase side effects and lead to
serotonin syndrome from too much serotonin
Symptoms of serotonin syndrome
Answer: • mild (shivering and diarrhoea)
• sever (muscle rigidity, fever, seizures)
• also tachycardia and hypertension
• also agitation/restlessness
Patient care for acute manic episode
Answer: • reduce environment stimuli
• calm quiet low lit
• limiting person contact
• ONE on ONE- not group activities
• structured schedule of activities to keep focus
• physical activities one on one to relieve energy
• high protein high calorie finger foods
• Set Limits
Signs of depression in adolescents
Answer: 1. Hypersomnolence or insomnia; napping during daily activities
2. Low self-esteem; withdrawal from previously enjoyable activities
3. Outbursts or angry, aggressive, or delinquent behavior (eg vandalism, absenteeism);
inappropriate sexual behavior
4. Weight gain or loss ; increased food intake or lack of interest in eating
Before a lumbar puncture teaching

Answer: empty bladder lateral recumbent (fetal position) or sitting
pain may be felt radiating down leg but should be temporary
After a LP teaching
Answer: • lie flat with no pillow for at least 4 hours
• increase fluid intake
What does normal CSF look like/contain?
Answer: colorless and odorless
A little protein, glucose, minimal WBC’s, no RBC’s, and no microorganisms digoxin function
Answer: increases cardiac contractility but slows heart rate excreted by the kidney
When is digoxin used?
Answer: in heart failure to increase cardiac output and in a fib to reduce the heart rate
What labs to check before giving digoxin?
Answer: Bun and creatinine because this drug is mostly excreted by the kidney
Who to watch for while taking digoxin?
Answer: elderly and those with kidney injury because they have decreased kidney function
and digoxin can build up so they must be monitored frequently
Early signs of dig toxicity
Answer: nausea and vomiting
Late signs of dig toxicity
Answer: arrhythmias
Since digoxin lowers the heart rate, it is important to check what before giving?
Answer: The heart rate. Make sure it is above 60 bpm
Fluoxetine (Prozac)
Answer: antidepressant SSRI

Contraindications to thrombolytics
Answer: • previous brain
• bleed
• ischemic stroke or head trauma within 3 month
• active bleeding
• cerebral aneurysm
All thrombolytics lyse the pathologic clot (one what’s causing the MI) and may lyse other
clots so bleeding is a complication
Answer: Yes, thrombolytics target the pathological clot causing the MI but may also lyse
other clots, increasing the risk of bleeding complications.
Having chest pain typical of acute MI for 6 hours or less and no absolute contraindications
you can then be given what
Answer: thrombolytic therapy
Watch ALS patients for swallowing and respiratory difficulty
Answer: Monitor ALS patients for swallowing and respiratory difficulties due to muscle
weakness.
What is dysarthria?
Answer: worsening ability to speak- can lead to respiratory distress
Meds should be stored specifically how when hiding from children?
Answer: out of sight up in a cabinet and locked
What should the nurse do if the chest tube becomes separated from drainage tubing and is not
contaminated?
Answer: wipe the end with antiseptic immediately reconnect it, and secure with tape
What about if a chest tube separates from the drainage tube and becomes contaminated?

Answer: submerge the distal end 1-2 cm below the surface of sterile water to create a water
seal so that air does not get in. keep bottle of sterile water, clamps, and antiseptic wipes at the
bedside
Clamping a chest tube should not happen for longer than a minute and only if prescribed by
the HCP
Answer: Clamping a chest tube should not exceed one minute and should only be done if
prescribed by the healthcare provider.
Characteristics of herpes zoster
Answer: unilateral linear pattern of fluid filled blisters patients report pain and itching
The varicella zoster virus can cause what 2 things?
Answer: chicken pox and shingles
Relation of varicella/chicken pox/herpes zoster
Answer: varicella causes chickenpox in childhood and then the virus goes dormant.
reactivation occurs with compromised immune system (eg aging or immunosuppression) can
get a vaccination to prevent shingles
Herpes Zoster (Shingles) Characteristics
Answer: Unilateral, linear (dermatomal distribution) vesicular rash (fluid filled blisters)
If a child under 1 years of age is choking, what do you do
Answer: back blows and chest thrusts
For adults and children over 1, how do you help someone that is choking
Answer: Heimlich maneuver (abdominal thrusts)
If the child swallowed something and is conscious and able to cough or make sounds, what
should the nurse do first
Answer: Have them try to cough it up, but signs of respiratory distress require immediate
intervention

What is hyperthyroidism?
Answer: increased T3 and T4 hormones which increases the metabolic rate
Symptoms of hyperthyroidism
Answer: everything is increased! like if someone is moving around a lot they would get all
sweating and tired and lose weight
• anxiety/insomnia
• palpitations
• heat intolerance sweating
• weight loss without decreased appetite
• goitre
• hypertension
• hand/finger tremors
• eyeball protrusion
What is hypothyroidism?
Answer: thyroid hormone deficiency, low T3 and T4 levels which decreases the metabolic
rate
Symptoms of hypothyroidism
Answer: • bradycardia
• dry skin
• weight gain
Signs of complications post tonsillectomy & adenoidectomy
Answer: Risk for haemorrhage up to 14 days after surgery and life threatening airway
compromise
Signs of Haemorrhage:
1. Restlessness
2. Frequent swallowing or clearing of throat
3. Vomiting of blood
4. Pallor
May require surgery to cauterize bleeding vessels

What are some common findings 5-10 days after a tonsillectomy?
Answer: slight ear pain, low grade fever, and bad breath
How to prevent haemorrhage after a tonsillectomy
Answer: avoid clearing the throat, blowing the nose, or coughing
What is pyrosis?
Answer: heartburn
Ises heartburn in pregnancy?
Answer: progesterone
How can pregnant women avoid heartburn?
Answer: • upright
• position after meals
• small frequent meals
• keep HOB elevated
•drink smaller amounts of fluid while eating
• eliminate dietary trigger like caffeine, chocolate, fried/fatty foods, spicy foods , peppermint
• do not take Alka seltzer while pregnant
Examples of proton pump inhibitors (PPIs)
Answer: "pyrazoles" omeprazole, lansoprazole, pantoprazole
Long-term use of PPIs can cause what?
Answer: Decreased bone density, which increases the risk for fractures, and can also cause
C. diff
Phenytoin toxicity
Answer: ataxia (gait disturbance)
What are soe ways iron deficiency can happen?
Answer: • diets low in iron like vegetarian
• iron not being absorbed like after abdomen surgery or malabsorption syndrome

• increased iron requirement blood loss
What are some iron rich foods?
Answer: • meats- beef, lamb, liver chicken, pork
• shellfish- oysters, clams, shrimp
• eggs, green leafy veggies, broccoli, dried fruits, dried beans, brown rice, oatmeal
• eating foods rich in vitamin c like citrus fruits potatoes and tomatoes and also orange juice
can enhance the absorption of iron
A nurse should listen to breath sounds of a client with heart failure related fluid overload
because they are likely to have dyspnoea, orthopnoea, and rales/crackles
Answer: A nurse should listen to the breath sounds of a client with heart failure due to fluid
overload, as they may exhibit dyspnoea, orthopnoea, and rales/crackles.
Fall risks
Answer: • 65 or above
• the use of assistive ambulatory devices like canes or crutches
• orthostasis
• taking sedatives or antiparkinsonian med like Sinemet (carbidopa/levodopa)
Drugs that cause hyperkalaemia
Answer: 1. Potassium sparing diuretics (spironolactone)
2. ACE inhibitors (-pril)
3. NSAIDs
Indicates of increased ICP
Answer: change in LOC (often the first sign), crushing’s triad, and pupillary changes)
If spine injury is suspected, do not use the head tilt chin lift method as this could damage the
spine further. what do you use instead?
Answer: jaw thrust maneuver
With any suspicion of spinal injury in a trauma, always use the jaw manuever

Answer: In cases of suspected spinal injury during trauma, always use the jaw thrust
maneuver to secure the airway without risking further injury.
Given what meds in hypovolemic shock (besides fluids) ?
Answer: norepinephrine and dopamine
What type of fluids would someone need while in hypovolemic shock?
Answer: isotonic like normal saline and lactated ringers
In cystic fibrosis, pancreatic ducts are blocked in the GI tract due to thick secretions that clog
them. therefore they are deficient in what type of enzymes?
Answer: pancreatic enzymes that are given with meals
Cystic fibrosis diet
Answer: high calorie, high fat, high protein
Kidney disorders diet
Answer: low phosphate
Heart failure, ascites, hypertension diet
Answer: low sodium (fluid overload states)
Antisocial personality disorder
Answer: • They disregard rules, have a history of irresponsible behavior, and blame others
for their actions.
• Set limits with them.
• Make them aware of the rules and acceptable behaviors
In pregnancy, pts are considered anaemic when haemoglobin drops below what?
Answer: 45 mmHg
Which lab results are expected for a patient with severe COPD: anaemia, neutropenia,
polycythaemia, or thrombocytopenia?
Answer: polycythaemia- patient is in hypoxemic state so body makes more RBCs to
compensate
Blood transfusions can only be run with what other fluid?
Answer: normal saline
What does a STAT order mean?
Answer: immediately and only once
The positive Babinski is normal in children under 1
Answer: positive means big toe moves up and other toes fan out
What does frostbite a actually men
Answer: •ice crystals form in the intracellular spaces causing peripheral vasoconstriction and
reduced
• blood flow
• superficial frostbite looks: mottled, blue, or waxy yellow
• deeper frostbite looks: white heard and unable to sense touch
• leads to gangrene
Treatment of frostbite
Answer: • remove clothing/jewellery to prevent vasoconstriction
•do not massage rub or squeeze he affected area
• immerse affected areas in warm water
• provide analgesia- rewarming is painful

• elevate affected areas
• let areas become dry then put loose nonadherent dressings
Expected number of stools per day in patient on lactulose
Answer: 2-3 soft stools per day
How to reduce portal pressure in liver failure and oesophageal varices
Answer: TIPS (trans jugular intrahepatic portosystemic shunt) procedure and beta blockers
(propranolol)
TIPS procedure is used for what?
Answer: oesophageal varices
What is spironolactone and what is it used in?
Answer: potassium sparing diuretic used in liver failure, ascites, and edema to promote
diuresis
EpiPen delivery
Answer: • can be delivered through clothing in the out thigh
• hold for 10 seconds so all med goes in then massage the thigh for 10 more seconds
When is IV epinephrine indicated?
Answer: profound hypotension or when a patient does not respond to IM epinephrine and
fluid resuscitation
Should bulimic patients be allowed to keep a door diary in the hospital?
Answer: yes, to keep track of the amount and type of food the client has eaten
The delivery if oxytocin
Answer: • given through an infusion
• pump into a proximal port (not distal)
•monitor FHR and uterine contraction pattern at least every 15 mins
• monitor intake and output
• continuous FHR monitoring

Be careful for giving too much oxytocin why
Answer: can lead to uterine tachysystole, decreased placental perfusion, and fetal distress.
this is why its given via an infusion pump so there are no med errors
Why do we monitor intake and output during oxytocin infusion?
Answer: can lead to water intoxication
Metoprolol
Answer: slows the HR and lowers the BP
Review heart strips and which meds to be given/actions to take when patients are in certain
heart rhythms
Answer: Review heart strips for rhythms like AFib (anticoagulants), VT (antiarrhythmics),
and VF (defibrillation), with actions based on hemodynamic stability and symptoms.
Hydrocodone/acetaminophen oral tablet is typically used for what type of pain?
Answer: moderate
Morphine is typically used for what type of pain?
Answer: severe
Remember when measuring fundal height, that after 20 weeks it correlates with number of
weeks pregnant. so if you are 24 weeks pregnant, the fundal height should be 24cm above the
pubis symphysis
Answer: After 20 weeks, fundal height in centimetres typically correlates with gestational
age in weeks, so at 24 weeks pregnant, the fundal height should measure about 24 cm above
the pubic symphysis.
What is quickening?
Answer: the awareness of fetal movement
When should quickening happen?
Answer: 18-20 weeks in first time baby havers and sooner for previous moms

First trimester in weeks
Answer: 1-12 weeks
Second trimester in weeks
Answer: 13-26 weeks
Third trimester in weeks
Answer: 27 weeks to end of pregnancy
How to calculate MAP?
Answer: 2xdiastolic + systolic all divided by 3
Proton pump inhibitors examples
Answer: • pantoprazole and omeprazole
• given for GERD, ulcer treatment and prophylaxis
IV proton pump inhibitors are given for
Answer: gastric ulcer bleeding
What antibiotic is used for C diff?
Answer: vancomycin
What are lab results that support a vaso-occlusive crisis (pain crisis) in sickle cell patients?
Answer: elevated reticulocytes
• elevated bilirubin
• anaemia (Hgb 100.4F (38 C)
2. productive or dry cough
3. changes in secretions
Ventricular bigeminy must be assessed rapidly because it can lead to What?
Answer: Deadly arrhythmias like ventricular tachycardia (V-tach) or ventricular fibrillation
(V-fib)
What is magnesium sulfate used for?
Answer: to stop seizures during pregnancy therapeutic level is 4-7
When is the rapid response team notified?
Answer: acute significant changes in heart rate, systolic BP, RR, O2 sat, LOC, and urine
output
Physical findings of peripheral arterial disease
Answer: weak/absent peripheral pulses, poor wound healing, cool pallid skin, shiny skin,
missing hair
What does failing the pinprick test indicate?
Answer: peripheral neuropathy

If there is a puncture wound to a person by an object, the nurse should not try to remove the
object. instead, they should stabilize it with dressings
Answer: this my cause further trauma and bleeding
Bumetanide (Bumex)
Answer: heart failure to diurese
When doing compressions on an infant, where do you place your fingers?
Answer: right below imaginary horizontal line between the nipples
Steps after newborn is born
Answer: stimulate and if after 30 seconds if hr remains <100 then start PPV. if after 30
seconds still reminds <60 after quality PPV then do chest compressions and if still <60 then
give epinephrine
What is the first line treated for Kawasaki disease?
Answer: IV immunoglobulin and aspirin to prevent the aneurism
Parent teaching for Kawasaki disease
Answer: check for fever
What is Kawasaki disease and what are the symptoms?
Answer: think of it as one big body rash
systemic vasculitis in children presenting with more than 5 day of a fever, conjunctivitis,
lymphadenopathy, mucositis, hand and foot swelling, an a rash, can cause coronary artery
aneurisms some skin peeling joint pain and irritability
A DNR does not provide direction for nutrition supplementation
Answer: A DNR (Do Not Resuscitate) order does not address nutrition supplementation
decisions.
What are some things that put patients at risk for respiratory depression?

Answer: Elderly age, underlying pulmonary conditions, history of snoring, obesity, cigarette
smoking, opiate-naive status, and post-surgery within 24 hours
What is acute RLQ pain, nausea/vomiting, and a high WBC count most likely?
Answer: appendicitis. emergency surgery because appendix could rupture. if it ruptures, the
pain will subside temporarily (until peritonitis/sepsis develops)
Gastric lavage "stomach pumped"
Answer: orogastric tube removes toxins and irrigates stomach rarely performed unless
overdose could be lethal and if it can be done within one hour of the overdose. not done a lot
because of high risk for aspiration or oesophageal/gastric perforation intubation/suction
should always be at bedside elevate HOB or put patient on side use large bore
What is the standard treatment for overdose?
Answer: charcoal (but its ineffective for lithium, iron, and alcohol)
Infant formula teaching
Answer: 1. keep parts of bottles as clean as possible (boil or wash in dishwasher)
2. wash the top of can before opening
3. keep unused prepared formula or cans in refrigerator after opening but discard after 48
hours but throw away formula that infant left in bottle
4. never microwave but instead can be placed in pan of hot water to warm it, then test temp
on inner wrist. should be lukewarm
5. Never dilute or concentrate formula
What is a magnetic resonance cholangiopancreatography
Answer: non-invasive diagnostic to see biliary, hepatic, and pancreatic ducts via MRI.
contrast (IV gadolinium) given before contraindications of the procedure that should be
assessed by nurse beforehand:
• any metal/electrical implants
• any allergic reaction to contrast
• pregnancy- the contrast crosses the placenta

Document Details

  • Subject: Nursing
  • Exam Authority: ATI
  • Semester/Year: 2023

Related Documents

person
Emma Thompson View profile
Close

Send listing report

highlight_off

You already reported this listing

The report is private and won't be shared with the owner

rotate_right
Close
rotate_right
Close

Send Message

image
Close

My favorites

image
Close

Application Form

image
Notifications visibility rotate_right Clear all Close close
image
image
arrow_left
arrow_right